Compleja:Zill-Cap5.3

De luz-wiki
Revisión del 18:21 14 jul 2015 de Ivan de Jesús Pompa García (discusión | contribs.) (Página creada con «Category:Compleja Ejercicios del capítulo 5, sección 3 del libro, A First Course in Complex Analysis with Applications de Zill y Shanahan. ---- ==Sección 5.3== ...»)
(difs.) ← Revisión anterior | Revisión actual (difs.) | Revisión siguiente → (difs.)


Ejercicios del capítulo 5, sección 3 del libro, A First Course in Complex Analysis with Applications de Zill y Shanahan.



Sección 5.3

Ejercicio 1

Muestre que $\oint_{C} \! f(z)\,dz=0$, donde $f(z)=z^3-1+3i$ y $C$ es la circunferencia unitaria $|z|=1$.

Naturalmente surgen dos formas de resolver este ejercicio, la primera, más directa es parametrizar la curva, evaluar la función e integrar; si bien es fácil pensar de esta forma puede se transparente, es sin duda laboriosa. La segunda, consiste en usar el teorema de Cauchy para asumir que el resultado es 0, bastando así comprobar as condiciones de cauchy para la función dentro del dominio, a continuación presento ambos razonamientos:

Parametrizando: \[ z(t)=\cos (t)+ i\,\sin (t)\;\;\;\;0\leq t \le 2\pi \] \[ dz=\left[-\sin (t)+ i\,\cos (t)\right]\, dt \] Entonces: \[ \oint_{C} \! f(z)\,dz=\int_{0}^{2 \pi} \! \left[ \left(\cos (t)+ i\,\sin (t)\right)^3 -1+3i\right]\left[-\sin (t)+ i\,\cos (t)\right]\, dt=\int_{0}^{2 \pi} \! \left[ \cos (3t) +i\,\sin(3t) -1+3i\right]\left[-\sin (t)+ i\,\cos (t)\right]\, dt \] \[ =\int_{0}^{2 \pi} \! \left[ (-3-i)\cos(t) + i\,\cos(t) \cos(3t) + (1-3i) \sin(t) -\cos(3t) \sin(t) -\cos(t) \sin(3t) -i\,\sin(t) \sin(3t) \right]\, dt=\int_{0}^{2 \pi} \! \left[ (-3-i)\cos(t) + i\,\cos(4t) + (1-3i)\sin[t]-\sin(4t)\right]\, dt \] \[ =(-3-i)\int_{0}^{2 \pi} \! \cos(t) \, dt+i\int_{0}^{2 \pi} \! \cos(4t) \, dt+(1-3i)\int_{0}^{2 \pi} \! \sin(t) \, dt-\int_{0}^{2 \pi} \! \sin(4t) \, dt \] Claramente todas estas integrales valen 0 pues se integra en multiplos del periodo de estas funciones trigonometricas. \[ \therefore \oint_{C} \! f(z)\,dz=(-3-i)(0)+i(0)+(1-3i)(0)-0=0+0+0+0=0 \blacksquare \]

De la otra forma, basta con verificar que $f$ sea analítica en el dominio. Bastaria con observar que $f$ esta formada por combinación de funciones elementales que son analíticas, pero a fin de qu este razonamiento sean contundente se procede del siguiente modo:

Descomponemos $f$: \[ f(z)=f(x+iy)=u(x,y)+i\,v(x,y)=z^3-1+3i=(x+iy)^3-1+3i=(x^3+3ix^2y-3xy^2-iy^3)-1+3i=(x^3-3xy^2-1)+i(3x^2y-y^3+3) \] \[ \therefore \; u(x,y)=x^3-3xy^2-1 \;\;\;\;\;\;v(x,y)=3x^2y-y^3+3 \] Para que $f$ sea analítica debe cumplir que: \[ \frac{\partial u}{\partial x}=\frac{\partial v}{\partial y}\;\;\;\;\;\;\;\;\frac{\partial v}{\partial x}=-\frac{\partial u}{\partial y} \] \[ \frac{\partial u}{\partial x}=\frac{\partial (x^3-3xy^2-1)}{\partial x}=3x^2-3y^2 \;\;\;\;\; \frac{\partial u}{\partial y}=\frac{\partial (x^3-3xy^2-1)}{\partial y}=-6xy \] \[ \frac{\partial v}{\partial x}=\frac{\partial (3x^2y-y^3+3)}{\partial x}=6xy \;\;\;\;\; \frac{\partial v}{\partial y}=\frac{\partial (3x^2y-y^3+3)}{\partial y}=3x^2-3y^2 \] Donde se verfica la analiticidad de $f$ para todo $\mathbb{C}$, y por lo tanto: \[ \therefore \oint_{C} \! f(z)\,dz=0 \blacksquare \]

--Tlacaelel Cruz (discusión) 22:57 16 jun 2015 (CDT)


Ejercicio 5

Mostrar que $\oint_{C}f(z)dz=0$, donde $f$ es una función dada y $C$ es la circunferencia unitaria $|z|=1$.

$f(z)=\frac{senz}{(z^{2}-25)(z^{2}+9)}$

Entonces:

$\oint_{C}f(z)dz=\oint_{C}\frac{senz}{(z^{2}-25)(z^{2}+9)}dz$

Vemos que tenemos sigunlaridades en:$z=\left\{ 5;-5;\right\} $ , sin embargo, estas no están dentro de la circunferencia $|z|=1$.

Por tanto, se puede demostrar que, efectivamente, la integral de contorno es igual a cero.

Se sigue por tanto que esta función es...

$\oint_{C}f(z)dz=\oint_{C}\frac{senz}{(z^{2}-25)(z^{2}+9)}dz=0$

Para ver que esto se cumple, y no dejarlo a la fé, resolviendo la integral...

Primero resolvemos por fracciones parciales como sigue

Factorizando como más convenga...

$\frac{senz}{(z^{2}-25)(z^{2}+9)}=\frac{senz}{(z-5i)(z-5i)(z+3)(z-3)}=\frac{senz}{(z-5i)^{2}(z+3)(z-3)}=\frac{Az+B}{z^{2}-25}+\frac{C}{(z-3)}+\frac{D}{(z+3)}$

Desarrollando nos queda de la forma:

$Az+B(z-3i)(z+3i)+C(z+3i)(z^{2}-25)+D(z-3i)(z^{2}-25)$

$=Az^{3}+9Az+Bz^{2}+9B+Cz^{3}+3Cz^{2}i+75Ci+Dz^{3}-3Dz^{2}i-25Dz+75Di$

Agrupando:

$(A+C+D)z^{3}+(B+3iC-3iD)z^{2}+(9A-25D)z+9B-75iC+75iD$

Entonces, igualando términos semajantes, formamos un sistema de ecuacionesde la siguiente manera:

$A+C+D=0$

$B+3iC-3iD=0$

$9A-25D=0$

$9B-75iC+75iD=senz$

Se puede comprobar que

$A=\frac{2}{43}senz$

$B=0$

$C=-\frac{1}{43}senz$

$D=\frac{senz}{43}$

Y con esto podemos resolver como sigue:

$\oint_{C}\frac{senz}{(z^{2}-25)(z^{2}+9)}dz=\oint_{C}\frac{senz}{(z^{2}-25)(z^{2}+9)}dz=\oint_{C}\frac{senz}{(z-5i)(z-5i)(z+3)(z-3)}dz=\oint_{C}\frac{senz}{(z-5i)^{2}(z+3)(z-3)}dz=\oint_{C}\frac{Az+B}{z^{2}-25}dz+\oint_{C}\frac{C}{(z-3)}dz+\oint_{C}\frac{D}{(z+3)}dz$

$=\frac{2}{43}\oint_{C}\frac{senz}{z^{2}-25}dz-\frac{1}{43}\oint_{C}\frac{senz}{(z-3)}dz-\frac{1}{43}\oint_{C}\frac{senz}{(z+3)}dz=0$

De aquí vemos que cada fracción parcial cumple con los criterios de la condición que se pide para que sea cero. Ya que se cumple que cada singularidad está fuera de la circunferencia de radio 1.


--A. Martín R. Rabelo (discusión) 19:08 21 jun 2015 (CDT)




Ejercicio 6

Ejercicio 6

$f\left(z\right)=\frac{e^{z}}{2z^{2}+11z+15}$ y $\left|z\right|=1$

Mostrar que:

$\oint_{c}f\left(z\right)dz=0$

$f\left(z\right)=\frac{e^{z}}{2z^{2}+11z+15}=\frac{e^{z}}{\left(2z+5\right)\left(z+3\right)}$ ...$\left(1\right)$

$f\left(z\right)$es analítica porque en el denominador tenemos una función polinomial $g\left(z\right)$que es

analítica porque $g\prime\left(z\right)=4z+11$y en el numerador tenemos $p\left(z\right)=e^{z}$que es analítica porque

$p\prime\left(z\right)=e^{z}$

Pero de la función $f\left(z\right)$ se puede ver que existen dos singularidades en

$z=-\frac{5}{2}$ y en $z=-3$

Pero ambas singularidades se encuentran fuera de C por lo que

$f\left(z\right)$sí es analítica dentro y sobre C.

Por lo que según el teorema de Cauchy-Goursat

$\oint_{c}f\left(z\right)dz=0$


Resuelto por:


--Alejandro Juárez Toribio (discusión) 13:36 18 jun 2015 (CDT)


Ejercicio 7

Muestre que $\oint_{C} \! f(z)\,dz=0$, donde $f$ es la funcion dada y $C$ es la circunferencia unitaria $|z|=1$


$f(z)=tanz$


Donde $z=cost+isint$ y $dz=(-sint+icost)dt$


Entonces tendríamos


$\oint_{0}^{2\pi} \! tan(cost+isint)\,(-sint+icost)dt$


Haciendo un cambio de variable


$u=cost+isint$ y $du=(-sint+icost)dt$


y si $t=0$ entonces $u=1$


y $t=2\pi$ entonces $u=1$, sustituimos valores


$\oint_{1}^{1} \! tan(u)\,du = -ln(cosu)\mid_{1}^{1}$


$\oint_{1}^{1} \! tan(u)\,du = -ln(cos1)+ln(cos1) $, entonces


$\oint_{1}^{1} \! tan(u)\,du = 0$


Miguel Medina Armendariz (discusión) 17:27 19 jun 2015 (CDT)


Ejercicio 9

Evaluate $\oint_{c}\frac{1}{Z}dz$ where c is the contour shown in Figure 5.34.

traduccion

evaluar $\oint_{c}\frac{1}{Z}dz$, Donde C es el contorno mostrado en la Figura 5.34 .

resolucion:

ppodriamos realisar la integral de linea sobre la curva c; pero resulta que c es una curva cerrada simple generada por los valores absolutos de X y de Y; por tanto usaremos el Teorema de Cauchy - Goursat el cual nos dice que si una fincion es analitica y esta avaluada sobre una superficie cerrada simple donde tiene una frontera c; entonces esta integral vale cero

por lo tanto verifiquemos si la funcion es analitica

$F\left(z\right)=\frac{1}{z}$

desarrollamos a f$\left(z\right)$ teniendo $z=x+iy$tenemos

$f\left(z\right)=\frac{1}{x+iy}=\frac{1}{x+iy}\left(\frac{x-iy}{x-iy}\right)=\frac{x-iy}{x^{2}+y^{2}}$

$poniendof\left(z\right)enfunciondeu\left(x,y\right)+iv\left(x,y\right)$

$U\left(x,y\right)=\frac{x}{x^{2}+y^{2}}$

$V\left(x,y\right)=\frac{-y}{x^{2}+y^{2}}$

haora que la tenemos de esa forma devemos verificar que se cumplan las condiciones de Cauchy- Riemann para lo cual devemos verificar que:

$\frac{\partial u}{\partial x}=\frac{\partial v}{\partial y}$ a la ves que $\frac{\partial u}{\partial y}=-\frac{\partial v}{\partial x}$

entonces calculemos:

$\frac{\partial u}{\partial x}=\frac{y^{2}-x^{2}}{\left(x^{2}+y^{2}\right)^{2}}$la otra parcial $\frac{\partial v}{\partial y}=\frac{y^{2}-x^{2}}{\left(x^{2}+y^{2}\right)^{2}}$las parciales son iguales por lo cual se probo la primer condicion de Cauchy- Riemann

probemos la segunda:

$\frac{\partial u}{\partial y}=\frac{-2xy}{\left(x^{2}+y^{2}\right)^{2}}$la otra parcial $\frac{\partial v}{\partial x}=\frac{2xy}{\left(x^{2}+y^{2}\right)^{2}}$ las parciales son iguales pero de signo distinto por lo cual comprobamos que la cuncion cumple con las dos condiciones de Cauchy- Riemann por lo cual la funcion es analitica

como la funcion es analitica y esta evaluada sobre una curva cerrada simple entonces concluimos que:

$\oint_{c}\frac{1}{Z}dz=0$


--Martin Flores Molina (discusión) 13:05 15 mayo 2015 (CDT) ----



Ejercicio 9 bis

correcccion:

evaluar la siguiente integral $\oint_{c}\frac{1}{Z}dz$ en la region encontrada dentro de valor absuluto de x valor absoluto de y

en el ejericicio anterior esta bien todo esepto en resultado ya que existe un teorema que dice lo siguiente

$\oint_{c}\frac{1}{\left(z+z_{0}\right)^{n}}dz$ donde z$_{0}$es cualquier numero complejo constante y c es cualquier region cerrada en el plano complejo,, este integral tiene siempre dos soluciones la solucion sera $\oint_{c}\frac{1}{\left(z+z_{0}\right)^{n}}dz=0$ si y solo si n es distinto de 1

y sera $\oint_{c}\frac{1}{\left(z+z_{0}\right)^{n}}dz=2ipi$ si y solo si n=1

entocnes si reescribimos la integral del ejercicio de la siguiente manera:

$\oint_{c}\frac{1}{\left(z+0\right)^{1}}dz$ esto es posible pues z$_{0}$puede ser cualquier constante

entonces teniendo la integral resuelta por el teorema anterior de la siguiente manera:

$\oint_{c}\frac{1}{\left(z+z_{0}\right)^{1}}dz=2ipi$ puesto que n=1


--Martin Flores Molina (discusión) 13:05 15 mayo 2015 (CDT) ----




Ejercicio 11

Evalúa la integral a lo largo del contorno dado

$\oint (z + \frac{1}{z})dz$

$|z|=2$

Parametrizando el contorno tenemos $|z|=2$ por $z(t) = e^{2it}$ con $0 \leq 2\pi$ y $dz = 2\pi i e^{2it} dt$

Sustituyendo estos valores en la integral tenemos:

$\oint (z + \frac{1}{z})dz = \int_{0}^{2\pi} (e^{2it} + \frac{1}{e^{2it}})(2ie^{2it})dt$

$= \int _{0}^{2\pi} 2ie^{4it} dt + \int _{0}^{2\pi} \frac{2ie^{2it}}{e^{2it}}dt$

$= \int _{0}^{2\pi} 2ie^{4it} dt + \int _{0}^{2\pi} 2idt = 2i\int _{0}^{2\pi} e^{4it} dt + 2i\int _{0}^{2\pi} dt $

$= 2i\int _{0}^{2\pi}e^{4it} dt + 2i[2\pi - 0] _{0}^{2\pi} $

Haciendo un cambio de variable para resolver la primera integral con $u = 4it$ y $du = 4idt$, sustituyendo tendríamos:

$= \frac{2i}{4i} \int _{0}^{2\pi} e^{u} du + 2i[2\pi - 0] _{0}^{2\pi} $

$= \frac{1}{2} [e^{4it}] _{0}^{2\pi} + 4i\pi$

Para evaluar a la exponencial podemos expresarla de la forma: $e^{4it} = 4[\cos {t} + i\sin{t}]$, entonces tendríamos

$= \frac{1}{2} (4 - 4) + 4i\pi$

$= 4i\pi$

Siendo este nuestro resultado

Angelina Nohemi Mendoza Tavera (discusión) 13:33 20 jun 2015 (CDT)


Ejercicio 12

Utilice cualquiera de los resultados de esta seccion para evaluar la integral a lo largo del contorno indicado cerrado.

Error al representar (SVG (MathML puede ser habilitado mediante un plugin de navegador): respuesta no válida («Math extension cannot connect to Restbase.») del servidor «https://en.wikipedia.org/api/rest_v1/»:): \oint(z+\frac{1}{z²})dz; |z|=2


para analizar la integral anterior podemos hacer:

Error al representar (SVG (MathML puede ser habilitado mediante un plugin de navegador): respuesta no válida («Math extension cannot connect to Restbase.») del servidor «https://en.wikipedia.org/api/rest_v1/»:): \oint_{C}z dz +\oint_{C}\frac{1}{z²}dz

Analizando la primera integral y tomando f(z)=z, vemos que la función puede ser cualquier función entera,por lo cual es analítica en cualquier contorno cerrado, asi pues :

Analizando la segunda integral tenemos que nuestra funcion tiene una singularidad para , la cual pertenece al dominio.

\begin{equation} \oint_{C} \! \frac{1}{\left(z-z_{o}\right)^n} = \left\lbrace \begin{array}{ll} 2 \pi i & n=1\\ 0, & n\neq 1 \end{array} \right. \end{equation}

Tomando en cuenta para nuestra integral que y , y tomando en cuenta que existe una singularidad en la función, podemos obtener de la condicion anterior que : Error al representar (error de sintaxis): \oint_{C}\frac{1}{z²}dz=0

Por lo cual para esta integral. Error al representar (error de sintaxis): \oint(z+\frac{1}{z²})dz=0 --Anahi Limas (discusión) 21:06 21 jun 2015 (CDT)


Ejercicio 13

Evaluar la integral a lo largo del contorno indicado.

$ \oint_{C} \! \frac{z}{z^{2}-\pi^2} \,dz $

$|z|=3$

Consideramos la siguiente definición:

\begin{equation} \oint_{C} \! \frac{1}{\left(z-z_{o}\right)^n} = \left\lbrace \begin{array}{ll} 2 \pi i & n=1\\ 0, & n\neq 1 \end{array} \right. \end{equation} \[ \]

$Solución: $

Resolvemos la integral por el método de fracciones parciales

Factorizando el denominador tenemos:


${z^{2}-\pi^2} = (z+\pi)(z-\pi)$


$ \frac{z}{z^2-\pi^2}=\frac{A}{z+\pi}+\frac{B}{z-\pi}=\frac{(A+B)z+(B-A)\pi} {(2+\pi)(z-\pi)} $


$A=\frac{1}{2}$ , $B=\frac{1}{2}$


Entonces la integral queda:

\[ \oint_{C} \! \frac{z}{z^{2}-\pi^2} \,dz= \frac{1}{2} \oint_{C} \! \frac{1}{z+\pi}\,dz + \frac{1}{2} \oint_{C} \! \frac{1}{z-\pi} \,dz \]


De acuerdo al teorema de Cauchy-Goursat la integral no esta en el contorno, es analitica en todos los puntos,asi que es 0.


$ \oint_{C} \! \frac{z}{z^{2}-\pi^2} \,dz = 0 $


Nancy Martínez Durán (discusión) 09:28 18 jun 2015 (CDT)


Ejercicio 14

Integrar a lo largo del contorno

$\oint_{c}\frac{z-1}{z\left(z-i\right)\left(z-3i\right)}dz$ donde $\left|z-i\right|=\frac{1}{2}$

Solución:

Resolveremos la integra como fracciones parciales de la siguiente manera:

$\frac{z-1}{z\left(z-i\right)\left(z-3i\right)}=\frac{A}{z}+\frac{B}{z-i}+\frac{C}{z-3i}=\frac{A\left(z-i\right)\left(z-3i\right)+Bz\left(z-3i\right)+Cz\left(z-i\right)}{z\left(z-i\right)\left(z-3i\right)}$

Entoces:


$z-1=A\left(z-i\right)\left(z-3i\right)+Bz\left(z-3i\right)+Cz\left(z-1\right)$

Resolviendo el sistema obtenemos los siguientes valores:

$A=\frac{1}{3}$ , $B=\frac{i-1}{2}$ , $C=\frac{1-3i}{6}$

Por lo tanto solamente sustituimos:

$\oint_{c}\frac{z-1}{z\left(z-i\right)\left(z-3i\right)}=\frac{1}{3}\oint_{c}\frac{dz}{z}+\frac{1}{2}\oint_{c}\frac{\left(i-1\right)}{z-i}dz+\frac{1}{6}\oint_{c}\frac{\left(1-3i\right)}{z-3i}dz$

Pero sabemos por Cauchy-Goursat que el valor de la primera y tercer integral es cero por lo tanto nos enfocamos en la segunda integral. Pero también sabemos que el valor de esa integral es 2ipi, entonces tenemos los sig:

$\frac{1}{2}\oint_{c}\frac{\left(i-1\right)}{\left(z-1\right)}dz=\frac{i-1}{2}\oint_{c}\frac{dz}{z-1}=\frac{\left(i-1\right)}{2}\left(2\pi i\right)=-\pi\left(1+i\right)$

Por lo tanto:

$\oint_{c}\frac{z-1}{z\left(z-i\right)\left(z-3i\right)}dz=-\pi\left(1-i\right)$


Resuelto por Luis Enrique Martínez Valverde (discusión) 21:38 16 jun 2015 (CDT)



Ejercicio 15

Utilice cualquiera de los resultados de esta sección para evaluar la integral a lo largo del(los) contorno(s) indicado(s) cerrados(s).

$\oint_{c}\frac{2z+1}{z^{2}+z}dz$ con los contornos (a) $|z|=\frac{1}{2}$, (b) $|z|=2$ y (c) $|z-3i|=1$

Primero notamos que:

$\oint_{c}\frac{2z+1}{z^{2}+z}dz=\oint_{c}\frac{2z+1}{z(z+1)}dz$


Ahora por fracciones parciales

$\frac{2z+1}{z(z+1)}=\frac{A}{z}+\frac{B}{z+1}\Longrightarrow2z+1=A(z+1)+Bz$


Entonces $A=1$ y $B=1$, y asi tenemos que

$\oint_{c}\frac{2z+1}{z^{2}+z}dz=\oint_{c}\frac{1}{z}dz+\oint_{c}\frac{1}{z+1}dz$


Se observa que el denominador de la integral no es analitico en $z=0$ y $z=-1$


Para (a), el punto $z=0$ sí esta dentro de c, pero $z=-1$ no lo esta. Por ello:

$\oint_{c}\frac{1}{z}dz+\oint_{c}\frac{1}{z-(-1)}dz=2\pi i+0$

Finalmente

\[ (a)\longrightarrow\oint_{c}\frac{2z+1}{z^{2}+z}dz=2\pi i \]


Para (b), ambos puntos se encuentran dentro de c, entonces:

$\oint_{c}\frac{1}{z}dz+\oint_{c}\frac{1}{z-(-1)}dz=2\pi i+2\pi i$

Tenemos:

\[ (b)\longrightarrow\oint_{c}\frac{2z+1}{z^{2}+z}dz=4i\pi \]



Y para (c), ninguno de los puntos esta dentro, entonces:

\[ (c)\longrightarrow\oint_{c}\frac{2z+1}{z^{2}+z}dz=0 \]


--Fernando Vazquez V. (discusión) 19:58 18 jun 2015 (CDT)



Ejercicio 16

evalúe la integral a lo largo de los contornos indicados cerrados

16.- $\oint_{C}\frac{2z}{z^{2}+3}dz$; a) $\mid z\mid=1$ b) $\mid z-2i\mid=1$, c) $\mid z\mid=4$

$\oint_{C}\frac{2z}{z^{2}+3}dz=\oint_{C}\frac{2z}{(z+i\sqrt{3})(z-i\sqrt{3})}dz$

y por fracciones parciales:

$\oint_{C}\frac{2z}{(z+i\sqrt{3})(z-i\sqrt{3})}dz=\oint_{C}[\frac{A}{(z+i\sqrt{3})}+\frac{B}{(z-i\sqrt{3})}]dz=\oint_{C}[\frac{A(z-i\sqrt{3})+B(z+i\sqrt{3})}{(z+i\sqrt{3})(z-i\sqrt{3})}]dz\Longleftrightarrow2z=A(z-i\sqrt{3})+B(z+i\sqrt{3})$

resolviendo la ecuación para $A\;y\;B$ tenemos:

si

$z=-i\sqrt{3}$

entonces:

$-i2\sqrt{3}=-Ai2\sqrt{3}\Longleftrightarrow A=1$

si

$z=i\sqrt{3}$

entonces:

$i2\sqrt{3}=Bi2\sqrt{3}\Longleftrightarrow B=1$

de esto:

$\oint_{C}\frac{2z}{(z+i\sqrt{3})(z-i\sqrt{3})}dz=\oint_{C}[\frac{A}{(z+i\sqrt{3})}+\frac{B}{(z-i\sqrt{3})}]dz=\oint_{C}[\frac{1}{(z+i\sqrt{3})}+\frac{1}{(z-i\sqrt{3})}]dz=\oint_{C}\frac{1}{(z+i\sqrt{3})}dz+\oint_{C}\frac{1}{(z-i\sqrt{3})}dz$

$\oint_{C}\frac{2z}{(z+i\sqrt{3})(z-i\sqrt{3})}dz=\oint_{C}\frac{1}{(z+i\sqrt{3})}dz+\oint_{C}\frac{1}{(z-i\sqrt{3})}dz$

evaluando la integral a lo largo de a) $\mid z\mid=1$

como se puede ver $\oint_{C}\frac{2z}{(z+i\sqrt{3})(z-i\sqrt{3})}dz$ no es una función a analítica en $z=i\sqrt{3}\;y\;en\;z=-i\sqrt{3}$ y como estos puntos no se encuentran dentro del contorno $\mid z\mid=1$ entonces

es analítica en todo su dominio $D$ ,dentro y sobre $C$

así por el teorema de Cauchy-Coursat

$\oint_{C}\frac{1}{(z+i\sqrt{3})}dz=0$

$\oint_{C}\frac{1}{(z-i\sqrt{3})}dz=0$

$\Longrightarrow\oint_{C}\frac{2z}{(z+i\sqrt{3})(z-i\sqrt{3})}dz=0$

evaluando a lo largo de b) $\mid z-2i\mid=1$, (circunferencia de radio $1$ centrada en $2i$)

por el razonamiento anterior $z=-i\sqrt{3}$ no se encuentra dentro del contorno $\mid z-2i\mid=1$ y asi

$\oint_{C}\frac{1}{(z+i\sqrt{3})}dz=0$

pero $z=i\sqrt{3}$ se encuentra dentro del contorno, entonces:

$\oint_{C}\frac{1}{(z-i\sqrt{3})}dz=2\pi i$ esto se deduce de:

si $z_{0}$es cualquier numero complejo constante interior a cualquier contorno cerrado simple $C$, entonces para un entero n tenemos

$\oint_{c}\frac{dz}{(z-z_{0})^{n}}=\begin{cases} 2\pi i & n=1\\ 0 & n\neq1 \end{cases}$

entonces:

$\Longrightarrow\oint_{C}\frac{2z}{(z+i\sqrt{3})(z-i\sqrt{3})}dz=2\pi i$

evaluando a lo largo de c) $\mid z\mid=4$ (circunferencia de radio $4$ centrada en $0+0i$)

de lo anterior podemos concluir:

$\oint_{C}\frac{1}{(z-(-i\sqrt{3}))}dz=2\pi i$

$\oint_{C}\frac{1}{(z-i\sqrt{3})}dz=2\pi i$

así:

$\Longrightarrow\oint_{C}\frac{2z}{(z+i\sqrt{3})(z-i\sqrt{3})}dz=4\pi i$

--Francisco Medina Albino (discusión) 21:51 19 jun 2015 (CDT)



Ejercicio 17

Utilice cualquiera de los resultados de esta sección para evaluar la integral a lo largo del(los) contorno(s) indicado(s) cerrados(s).

$ \oint_{C} \! \frac{-3z+2}{z^{2}-8z+12} \,dz=0 $

$(a)$ $|z-5|=2$

$(b)$ $|z|=9$


\begin{equation} \oint_{C} \! \frac{1}{\left(z-z_{o}\right)^n} = \left\lbrace \begin{array}{ll} 2 \pi i & n=1\\ 0, & n\neq 1 \end{array} \right. \end{equation} \[ (6) \]

$Solución: $


$(a)$ $|z-5|=2$


Factorizando el denominador

$z^{2}-8z+12 = (z-6)(z-2)$ el integrando no es analítico en $z=6$ y en $z=2$. De estos dos puntos sólo $z=6$ se encuentra dentro del contorno $C$, que es una

circunferencia centrada en $z=5$ de $r= 2$.

Ahora por fracciones parciales

\[ \frac{-3z+2}{z^{2}-8z+12}=\frac{A}{z-6}+\frac{B}{z-2}=\frac{A(z-2)+B(z-6)}{(z-6)(z-2)}= \frac{(A+B)z-2A-6B}{(z-6)(z-2)} \] $z: A+B=-3$

$c: -2A-6B=2$

Resolviendo tengo que:

$A=-4$ , $B=1$


\[ \frac{-3z+2}{z^{2}-8z+12}=\frac{-4}{z-6}+\frac{1}{z-2} \]

y así,

\[ \oint_{C} \! \frac{-3z+2}{z^{2}-8z+12} \,dz=-4 \oint_{C} \! \frac{1}{z-6}\,dz + \oint_{C} \! \frac{1}{z-2} \,dz ...(1) \]

De acuerdo con el resultado $(6)$, la primera integral en $(1)$ tiene el valor de $2 \pi i$, mientras que por el teorema de Cauchy-Gousart el valor de la segunda integral es 0. Por tanto $(1)$, se convierte en

\[ \oint_{C} \! \frac{-3z+2}{z^{2}-8z+12} \,dz= -4(2 \pi)i + 1(0)= -8 \pi i. \]


$(b)$ $|z|=9$


Factorizando el denominador

$z^{2}-8z+12 = (z-6)(z-2)$ el integrando no es analítico en $z=6$ y en $z=2$. Los dos puntos se encuentran dentro del contorno $C$, que es una circunferencia centrada en $z=0$ de $r= 9$.


y así,

\[ \oint_{C} \! \frac{-3z+2}{z^{2}-8z+12} \,dz=-4 \oint_{C} \! \frac{1}{z-6}\,dz + \oint_{C} \! \frac{1}{z-2} \,dz ...(2) \]

De acuerdo con el resultado $(6)$, las dos integrales en $(2)$ tiene el valor de $2 \pi i$. Por tanto, se convierte en

\[ -4 (2 \pi)i + 1(2 \pi)i =-8( \pi )i+(2 \pi) i = - 6 \pi i. \]


--Emmanuell Castro Flores (discusión) 20:45 17 jun 2015 (CDT)



Ejercicio 18

Calcular la integral a lo largo de los contornos indicados

a)

Analizando la función tenemos que las dos partes de la integral no son analíticas en puntos del dominio de la circunferencia de radio 5, es decir que

La función no es analitica en y además en y ambos puntos están dentro del dominio de la circunferencia de radio 5.

Por lo que procederemos a formar dos contornos circulares al rededor de cada punto respectivamente

en donde

en donde

con C1 que va en el intervalo de y C2 que va en el intervalo

Por lo que la integral queda definida como

Otra forma de ver es, dado si esto si hay puntos en donde la función no es analítica el dominio del contorno. Por lo mismo tenemos que



b)

Analizando el contorno y la funcion

únicamente la primera parte es analítica en el dominio dado que está fuera del dominio de la región, por otra parte, la segunda fracción de la función no es analítica en dicha región.

Tenemos por Cauchy-Goursat que

y por si , tenemos

por lo que para el segundo contorno el resultado es



--Pablo (discusión) 10:37 21 jun 2015 (CDT)




Ejercicio 23

Evalue : , donde C es la “forma de ocho” del contorno de la figura 5.3.11.

[Sugerencia. Exprese C como la unión de dos curvas cerradas C1 y C2]

Solución

Resolviendo la integral por fracciones parciales se tiene:

entonces:

Sustituyendo (2) en (1), y despejando “B” esta ultima que:

por lo tanto

Para la primera integral C1, z=0 esta dentro del contorno, y tiene valor de 2pi i; la segunda integral z =1

no esta dentro del contorno, no es analitica,asi que es 0;

para C2 la primera integral no esta dentro del contorno y es 0 ,la segunda integral por medio el teorema

de Cauchy-Goursat, la segunda integral esta dentro del contorno y tiene valor de 2pi i; simplificando se tiene:

por lo tanto:

Elaborado por Ricardo Garcia Hernandez--Ricardo Garcia Hernandez (discusión) 23:33 17 jun 2015 (CDT)


Ejercicio 24

Supongamos que $z_{0}$ es cualquier numero complejo constante interior a cualquier curva simple cerrada del contorno C. Muestre que para un entero positivo n.

\[ \oint_{c}\dfrac{dz}{(z-z_{0})^{n}} = \left\lbrace \begin{array}{ll} n= 1 & 2\pi i\\ n>1 & 0 \end{array} \right.\]


Tomamos como cualquier curva cerrada a una circunferencia centrada en $z=z_{0}$ y a $\gamma$ como otra curva que aísla a $z_{0}$ con un radio $|z-z_{0}|= R$. Entonces, si hacemos esto $z-z_{0}= \overline{z} = Re^{i \theta}$. Utilizando el teorema de Cauchy.

\[ \oint_{c}\dfrac{dz}{z-z_{0}} = \int^{2\pi}_{o} \dfrac{Re^{i\theta}}{Re^{i\theta}}= i \int^{2\pi}_{0}d \theta = 2i \pi\]


Si $n>1$ esto implica que $z_{0}$ está fuera de la curva por tanto la función es analítica y se cumple el teorema de Cauchy.

\[ \oint_{c} f(z) dz = 0\]

--Esther Sarai (discusión) 20:32 18 jun 2015 (CDT)Esther Sarai

Ejercicio 30

Evalúe $\int_Ce^zdz$ de $z=0$ a $z=2+2i$ sobre el contorno $C$ de la figura 5.3.12 (consultar el libro) que consiste en la recta $y=x$ y una circunferencia tangente a la recta en $(1,1)$.


Sol. (Se omite la figura, para mayor claridad consultar la imagen en el libro).

La curva $C$ esta dada en 3 segmentos $C_1$, $C_2$ y $C_3$. Se procede primero a calcular la integral de contorno sobre $C_1$.


Se tiene que la integral de contorno es $\int_Cf(z)dz=\int_a^bf(z(t))z'(t)dt$. Con $f(z)=e^z=e^x(\cos y+i\sin y)$, la curva sobre la que se integra es la recta $y=x$. Sea $x=t$ con $t\epsilon [0,1]$ (de la figura) como el parámetro, por lo tanto $y=t$ y si $z=x+iy=t+it=z(t)$, entonces $z'(t)=1+i$. Sustituyendo en la definición de la integral:


$\int_{C_1}f(z)dz=\int_a^bf(z(t))z'(t)dt=\int_0^1e^t(\cos t+i\sin t)(1+i)dt=\int_0^1e^t(\cos t+i\cos t+i\sin t-\sin t)dt=\int_0^1e^t(\cos t-\sin t)+i\int_0^1e^t(\cos t +\sin t)dt$


$\int_{C_1}f(z)dz=\int_0^1(e^t\cos t-e^t\sin t)+i\int_0^1(e^t\cos t +e^t\sin t)dt$


Las integrales se obtienen de tablas y son:


$\int e^x\cos xdx=\frac{1}{2}e^x(\sin x+\cos x)$ y $\int e^x\sin xdx=\frac{1}{2}e^x(\sin x-\cos x)$


Sustituyendo:


$\int_{C_1}f(z)dz=\frac{1}{2}[e^t(\sin t+\cos t)-e^t(\sin t-\cos t)]_0^1+i\frac{1}{2}[e^t(\sin t+\cos t)+e^t(\sin t-\cos t)]_0^1$


factorizando $e^t$ en cada termino y posteriormente sumanos y restamos:


$\int_{C_1}f(z)dz=\frac{1}{2}(2e^t\cos t)|_0^1+i\frac{1}{2}(2e^t\sin t)|_0^1=(e\cos 1-1)+i(e\sin 1)$


Así $\int_{C_1}f(z)dz=(e\cos 1-1)+i(e\sin 1)$


Para la curva $C_2$ que corresponde a la circunferencia tangente a la recta en $(1,1)$ se toma en cuenta que la función $f(z)=e^z$ es una función analítica en todo el plano complejo, lo cual es fácil comprobarse con el criterio del Teorema de Cauchy-Riemann. Dado lo anterior y del Teorema de Cauchy-Goursat, se tiene que la integral sobre el contorno cerrado $C_2$ es cero


$\int_{C_2}f(z)dz=0$


Para la curva $C_3$ se procede de igual forma que para $C_1$, sólo que los límites de la integral ésta vez cambian dado que el parámetro $t$ ahora se define para $t\epsilon [1,2]$. Así, simplemente sustituyendo límites:


$\int_{C_3}f(z)dz=(e^t\cos t)|_1^2+i(e^t\sin t)|_1^2=[e^2\cos 2-e\cos 1]+i[e^2\sin 2-e\sin 1]$


$\int_{C_3}f(z)dz=[e^2\cos 2-e\cos 1]+i[e^2\sin 2-e\sin 1]$


Se tiene que para la integral sobre el contorno $C$ conformada por los segmentos $C_1$, $C_2$ y $C_3$ es


$\int_Cf(z)dz=\int_{C_1}f(z)dz+\int_{C_2}f(z)dz+\int_{C_3}f(z)dz$


Sustituyendo cantidades:


$\int_Cf(z)dz=(e\cos 1-1)+i(e\sin 1)+(e^2\cos 2-e\cos 1)+i(e^2\sin 2-e\sin 1)=(e\cos 1-1+e^2\cos 2-e\cos 1)+i(e\sin 1+e^2\sin 2-e\sin 1)$


Por lo tanto:


$\int_Cf(z)dz=(e^2\cos 2-1)+i(e^2\sin 2)$


Oscar Javier Gutierrez Varela (discusión) 19:01 16 jun 2015 (CDT)


Ejercicio 31

En particular use $|z|=1$como $C$ y la parametrizacion $z=e^{i\theta}$,$0\leq\theta\leq2\pi,$para determinar el valor de las inegrales reales.

\[ {\displaystyle \int_{0}^{2\pi}e^{\cos\theta}\sin(\theta+\sin\theta)d\theta}={\displaystyle \frac{1}{2i}{\displaystyle \int_{0}^{2\pi}e^{\cos\theta}(e^{i(\theta+\sin\theta)}-e^{-i(\theta+\sin\theta)}d\theta}=}{\displaystyle \frac{1}{2i}{\displaystyle \int_{0}^{2\pi}e^{(\cos\theta+i(\theta+\sin\theta))}-e^{(\cos\theta-i(\theta+\sin\theta))}}} \]


\[ \Rightarrow{\displaystyle {\displaystyle \frac{1}{2i}}\int_{0}^{2\pi}e^{(\cos\theta+i\sin\theta)+i\theta}-e^{(\cos\theta-i\sin\theta)-i\theta}d\theta={\displaystyle \frac{1}{2i}}}{\displaystyle \int_{0}^{2\pi}{\displaystyle e^{e^{i\theta}}e^{i\theta}-e^{e^{-i\theta}}e^{-i\theta}d\theta}} \]


Usando $z=e^{i\theta}\Rightarrow d\theta={\displaystyle \frac{dz}{ie^{i\theta}}={\displaystyle \frac{dz}{iz}}}$ tenemos que

\[ {\displaystyle \frac{1}{2i}}\int_{0}^{2\pi}{\displaystyle e^{e^{i\theta}}e^{i\theta}-e^{e^{-i\theta}}e^{-i\theta}d\theta={\displaystyle \frac{1}{2i}\int_{a}^{b}\left(e^{z}z-{\displaystyle \frac{e^{1/z}}{z}}\right)\frac{dz}{iz}=\frac{1}{2}{\displaystyle \int_{a}^{b}(-e^{z}+e^{z^{-1}}z^{-2})dz}}} \]


\[ \Rightarrow{\displaystyle \frac{1}{2}}{\displaystyle \int_{a}^{b}}(-e^{z}+e^{z^{-1}}z^{-2})dz={\displaystyle -\frac{1}{2}\left(e^{z^{-1}}+e^{z}\right)|_{a}^{b}=-{\displaystyle \frac{1}{2}(e^{(\cos\theta-i\sin\theta)}+e^{(\cos\theta+i\sin\theta)})|_{0}^{2\pi}}}=0 \]


\[ \therefore\int_{0}^{2\pi}e^{\cos\theta}\sin(\theta+\sin\theta)d\theta=0 \]

Jose Emmanuel Flores Calderón (discusión) 19:43 21 jun 2015 (CDT)